Find all positive integers a,b,c,d such that $2019^a = b^3+c^3+d^3-5$












1












$begingroup$


Working on a problem...



Find all positive integers a,b,c,d such that $2019^a = b^3+c^3+d^3-5$



Using $(a+b+c)^3$ yields far too many terms, and I cannot come up with any solutions other than guessing and checking.










share|cite|improve this question









$endgroup$

















    1












    $begingroup$


    Working on a problem...



    Find all positive integers a,b,c,d such that $2019^a = b^3+c^3+d^3-5$



    Using $(a+b+c)^3$ yields far too many terms, and I cannot come up with any solutions other than guessing and checking.










    share|cite|improve this question









    $endgroup$















      1












      1








      1





      $begingroup$


      Working on a problem...



      Find all positive integers a,b,c,d such that $2019^a = b^3+c^3+d^3-5$



      Using $(a+b+c)^3$ yields far too many terms, and I cannot come up with any solutions other than guessing and checking.










      share|cite|improve this question









      $endgroup$




      Working on a problem...



      Find all positive integers a,b,c,d such that $2019^a = b^3+c^3+d^3-5$



      Using $(a+b+c)^3$ yields far too many terms, and I cannot come up with any solutions other than guessing and checking.







      contest-math






      share|cite|improve this question













      share|cite|improve this question











      share|cite|improve this question




      share|cite|improve this question










      asked Jan 10 at 4:56









      Hubert LioniHubert Lioni

      233




      233






















          1 Answer
          1






          active

          oldest

          votes


















          3












          $begingroup$

          HINT: If $a ge 2$, then $2019^a equiv 0 pmod{9}$, while $b^3 equiv -1, 0, text{or} 1 pmod{9}$ and similarly for $c^3$ and $d^3$.



          Do you see a contradiction? After ruling out $a ge 2$, you only need to solve the $a = 1$ case, i.e. $b^3+c^3+d^3 = 2024$. Since $b,c,d$ are positive integers, none of them can be larger than $sqrt[3]{2024}$, so there aren't too many possibilities.






          share|cite|improve this answer









          $endgroup$









          • 1




            $begingroup$
            Interesting, I solved a combination (10,8,8) earlier and wondered if this triple (and its permutations) are the only solutions. By your logic, they should be.
            $endgroup$
            – Hubert Lioni
            Jan 10 at 6:05













          Your Answer





          StackExchange.ifUsing("editor", function () {
          return StackExchange.using("mathjaxEditing", function () {
          StackExchange.MarkdownEditor.creationCallbacks.add(function (editor, postfix) {
          StackExchange.mathjaxEditing.prepareWmdForMathJax(editor, postfix, [["$", "$"], ["\\(","\\)"]]);
          });
          });
          }, "mathjax-editing");

          StackExchange.ready(function() {
          var channelOptions = {
          tags: "".split(" "),
          id: "69"
          };
          initTagRenderer("".split(" "), "".split(" "), channelOptions);

          StackExchange.using("externalEditor", function() {
          // Have to fire editor after snippets, if snippets enabled
          if (StackExchange.settings.snippets.snippetsEnabled) {
          StackExchange.using("snippets", function() {
          createEditor();
          });
          }
          else {
          createEditor();
          }
          });

          function createEditor() {
          StackExchange.prepareEditor({
          heartbeatType: 'answer',
          autoActivateHeartbeat: false,
          convertImagesToLinks: true,
          noModals: true,
          showLowRepImageUploadWarning: true,
          reputationToPostImages: 10,
          bindNavPrevention: true,
          postfix: "",
          imageUploader: {
          brandingHtml: "Powered by u003ca class="icon-imgur-white" href="https://imgur.com/"u003eu003c/au003e",
          contentPolicyHtml: "User contributions licensed under u003ca href="https://creativecommons.org/licenses/by-sa/3.0/"u003ecc by-sa 3.0 with attribution requiredu003c/au003e u003ca href="https://stackoverflow.com/legal/content-policy"u003e(content policy)u003c/au003e",
          allowUrls: true
          },
          noCode: true, onDemand: true,
          discardSelector: ".discard-answer"
          ,immediatelyShowMarkdownHelp:true
          });


          }
          });














          draft saved

          draft discarded


















          StackExchange.ready(
          function () {
          StackExchange.openid.initPostLogin('.new-post-login', 'https%3a%2f%2fmath.stackexchange.com%2fquestions%2f3068255%2ffind-all-positive-integers-a-b-c-d-such-that-2019a-b3c3d3-5%23new-answer', 'question_page');
          }
          );

          Post as a guest















          Required, but never shown

























          1 Answer
          1






          active

          oldest

          votes








          1 Answer
          1






          active

          oldest

          votes









          active

          oldest

          votes






          active

          oldest

          votes









          3












          $begingroup$

          HINT: If $a ge 2$, then $2019^a equiv 0 pmod{9}$, while $b^3 equiv -1, 0, text{or} 1 pmod{9}$ and similarly for $c^3$ and $d^3$.



          Do you see a contradiction? After ruling out $a ge 2$, you only need to solve the $a = 1$ case, i.e. $b^3+c^3+d^3 = 2024$. Since $b,c,d$ are positive integers, none of them can be larger than $sqrt[3]{2024}$, so there aren't too many possibilities.






          share|cite|improve this answer









          $endgroup$









          • 1




            $begingroup$
            Interesting, I solved a combination (10,8,8) earlier and wondered if this triple (and its permutations) are the only solutions. By your logic, they should be.
            $endgroup$
            – Hubert Lioni
            Jan 10 at 6:05


















          3












          $begingroup$

          HINT: If $a ge 2$, then $2019^a equiv 0 pmod{9}$, while $b^3 equiv -1, 0, text{or} 1 pmod{9}$ and similarly for $c^3$ and $d^3$.



          Do you see a contradiction? After ruling out $a ge 2$, you only need to solve the $a = 1$ case, i.e. $b^3+c^3+d^3 = 2024$. Since $b,c,d$ are positive integers, none of them can be larger than $sqrt[3]{2024}$, so there aren't too many possibilities.






          share|cite|improve this answer









          $endgroup$









          • 1




            $begingroup$
            Interesting, I solved a combination (10,8,8) earlier and wondered if this triple (and its permutations) are the only solutions. By your logic, they should be.
            $endgroup$
            – Hubert Lioni
            Jan 10 at 6:05
















          3












          3








          3





          $begingroup$

          HINT: If $a ge 2$, then $2019^a equiv 0 pmod{9}$, while $b^3 equiv -1, 0, text{or} 1 pmod{9}$ and similarly for $c^3$ and $d^3$.



          Do you see a contradiction? After ruling out $a ge 2$, you only need to solve the $a = 1$ case, i.e. $b^3+c^3+d^3 = 2024$. Since $b,c,d$ are positive integers, none of them can be larger than $sqrt[3]{2024}$, so there aren't too many possibilities.






          share|cite|improve this answer









          $endgroup$



          HINT: If $a ge 2$, then $2019^a equiv 0 pmod{9}$, while $b^3 equiv -1, 0, text{or} 1 pmod{9}$ and similarly for $c^3$ and $d^3$.



          Do you see a contradiction? After ruling out $a ge 2$, you only need to solve the $a = 1$ case, i.e. $b^3+c^3+d^3 = 2024$. Since $b,c,d$ are positive integers, none of them can be larger than $sqrt[3]{2024}$, so there aren't too many possibilities.







          share|cite|improve this answer












          share|cite|improve this answer



          share|cite|improve this answer










          answered Jan 10 at 5:18









          JimmyK4542JimmyK4542

          41.2k245107




          41.2k245107








          • 1




            $begingroup$
            Interesting, I solved a combination (10,8,8) earlier and wondered if this triple (and its permutations) are the only solutions. By your logic, they should be.
            $endgroup$
            – Hubert Lioni
            Jan 10 at 6:05
















          • 1




            $begingroup$
            Interesting, I solved a combination (10,8,8) earlier and wondered if this triple (and its permutations) are the only solutions. By your logic, they should be.
            $endgroup$
            – Hubert Lioni
            Jan 10 at 6:05










          1




          1




          $begingroup$
          Interesting, I solved a combination (10,8,8) earlier and wondered if this triple (and its permutations) are the only solutions. By your logic, they should be.
          $endgroup$
          – Hubert Lioni
          Jan 10 at 6:05






          $begingroup$
          Interesting, I solved a combination (10,8,8) earlier and wondered if this triple (and its permutations) are the only solutions. By your logic, they should be.
          $endgroup$
          – Hubert Lioni
          Jan 10 at 6:05




















          draft saved

          draft discarded




















































          Thanks for contributing an answer to Mathematics Stack Exchange!


          • Please be sure to answer the question. Provide details and share your research!

          But avoid



          • Asking for help, clarification, or responding to other answers.

          • Making statements based on opinion; back them up with references or personal experience.


          Use MathJax to format equations. MathJax reference.


          To learn more, see our tips on writing great answers.




          draft saved


          draft discarded














          StackExchange.ready(
          function () {
          StackExchange.openid.initPostLogin('.new-post-login', 'https%3a%2f%2fmath.stackexchange.com%2fquestions%2f3068255%2ffind-all-positive-integers-a-b-c-d-such-that-2019a-b3c3d3-5%23new-answer', 'question_page');
          }
          );

          Post as a guest















          Required, but never shown





















































          Required, but never shown














          Required, but never shown












          Required, but never shown







          Required, but never shown

































          Required, but never shown














          Required, but never shown












          Required, but never shown







          Required, but never shown







          Popular posts from this blog

          Human spaceflight

          Can not write log (Is /dev/pts mounted?) - openpty in Ubuntu-on-Windows?

          File:DeusFollowingSea.jpg